User avatar
 
ohthatpatrick
Thanks Received: 3808
Atticus Finch
Atticus Finch
 
Posts: 4661
Joined: April 01st, 2011
 
This post thanked 3 times.
 
 

Re: Q12 - Critic: A novel cannot be

by ohthatpatrick Fri Dec 31, 1999 8:00 pm

Question Type:
Necessary Assumption

Stimulus Breakdown:
Conclusion: Shifting POV in a novel detracts from the merit of the work.
Evidence: Shifting POV tends to make readers focus on the author. And a novel can't be of the highest quality if readers don't become emotionally engaged with the book's imaginary world.

Answer Anticipation:
This conclusion doesn't have any New Guys. Both of its ideas are Overlapping in the Evidence. We know that shifts of POV "make reader focus on author". We know that "not being emotionally engaged with a book's imaginary world" detracts from the merit of the book (because it can no longer be of the highest quality). So since the author is connecting shifts of POV to detracting from the work's merit, we know the author is trying to get from "reader focuses on author" to "reader can't get emotionally engaged with the book's imaginary world".

Correct Answer:
C

Answer Choice Analysis:
(A) Extreme = "most", "only if". This is really a backwards conditional. The author gave us the conditional that "highest quality -> emotionally engaged". This answer choice says "emotionally engaged -> highest quality".

(B) This says "high quality -> engaged imagination".
The argument gave us a rule for "highest quality". The author doesn't have to assume anything about "high quality". Also, we were talking about engaging EMOTIONS with the book's imaginary world, whereas this answer is talking about engaging OUR IMAGINATION. That's a detail creep.

(C) Yes! The author assumes that if we focus on the author, we won't get lost in the book's world. That was the connective tissue that was going to get the author from "shift in POV" to "detracting from merit of work".

(D) Out of scope = "who most readers regard as a novel's point of view". We were talking about when a book shifts points of view and how it makes the readers notice the author. You can't infer from that that the readers notice the author BECAUSE they think the novel's author represents the novel's narrative point of view. They might simply notice the author just because they're thinking, "ugh. this is annoying: the book just switched from third person to first person. Why would the author do that?"

(E) Extreme = "serve NO purpose". Even though the author is saying something negative about shifts of POV, these shifts can still serve a purpose. They just happen to cause the reader to notice the author and thus lower the quality of the work.

Takeaway/Pattern: The conclusion contained two ideas "shifting POV" and "detracting from merit". Both of those ideas were connected to something else in the evidence, and our job was to patch together those connected ideas. If this were algebra, and you knew that a = b and x = y, what would you need to know in order to prove that "a = x"? You would need to know that "b = y".

#officialexplanation
 
chike_eze
Thanks Received: 94
Atticus Finch
Atticus Finch
 
Posts: 279
Joined: January 22nd, 2011
 
 
trophy
Most Thanked
 

Q12 - Critic: A novel cannot be

by chike_eze Sun Oct 30, 2011 12:58 am

I approached this question as a Sufficient question type. Though I was initially confused by the use of the word "necessary" in the prompt.

Correct Answer (C)

Novel cannot be of Highest quality unless most readers are emotionally engaged with imaginary world...
- not Emotionally Engaged --> not High Quality
- High quality --> Emotionally Engaged

Shifts of narrative point tend to make readers focus on author -- which detracts from work (quality of work)

Assumption? shifts of narrative point -- leading to focus on author means readers are not emotionally engaged...

C) focus on author --> not emotionally involved (engaged)

A) Emotionally engaged --> High quality. No! illegal reversal.
B) "engages imagination". No! emotional engagement.
D) "novel's point of view represents author's perspective". Even if true, does not bridge gap between focus on author and not emotional engagement
E) "Shifts serve no literary purpose" -- not relevant to gap in argument
User avatar
 
maryadkins
Thanks Received: 641
Atticus Finch
Atticus Finch
 
Posts: 1261
Joined: March 23rd, 2011
 
 
 

Re: Q12 - Critic: A novel cannot be

by maryadkins Tue Nov 01, 2011 10:01 am

Nicely done! Totally on point for all of it.

One note: It's still a necessary assumption question. Can you see why?
 
chike_eze
Thanks Received: 94
Atticus Finch
Atticus Finch
 
Posts: 279
Joined: January 22nd, 2011
 
 
trophy
Most Thanked
 

Re: Q12 - Critic: A novel cannot be

by chike_eze Tue Nov 01, 2011 10:04 pm

@maryadkins, please explain - cos I guess if LSAT wanted to be a real pain in the a#$, they could have included a necessary and sufficient answer choice.
(or did they?)

Also, could you modify the question stem to read like a sufficient question type?
User avatar
 
maryadkins
Thanks Received: 641
Atticus Finch
Atticus Finch
 
Posts: 1261
Joined: March 23rd, 2011
 
 
 

Re: Q12 - Critic: A novel cannot be

by maryadkins Thu Nov 10, 2011 5:50 pm

Necessary answer choices can be sufficient too. This shouldn't throw you though. Because it's still true that negating it destroys the argument; that's all you need to worry about. In this case, negating (C) destroys the argument. (C) says:

Most readers cannot become emotionally
involved with a novel’s imaginary world if
they focus on the author.

So what if we negate it:

Most readers can become emotionally
involved with a novel’s imaginary world if
they focus on the author.

The argument doesn't work now!

chike_eze Wrote:could you modify the question stem to read like a sufficient question type?


If it were a sufficient assumption question, it would not have any synonym for "necessary" or "needs" in the stem (requires, depends on, needs, etc.). It would say "enables" or "allows" or "follows logically."
 
mkd000
Thanks Received: 0
Jackie Chiles
Jackie Chiles
 
Posts: 38
Joined: March 14th, 2015
 
 
 

Re: Q12 - Critic: A novel cannot be

by mkd000 Thu May 14, 2015 5:33 pm

I got this question correct but reviewing the question, I'm wondering how (A) can be considered a reversal. The first sentence of the stimulus says "most readers" so is it actually possible to call this sentence a conditional statement?

Please clarify! Thank you.
 
Antnat
Thanks Received: 0
Vinny Gambini
Vinny Gambini
 
Posts: 12
Joined: July 07th, 2015
 
 
 

Re: Q12 - Critic: A novel cannot be

by Antnat Wed Jul 22, 2015 5:36 am

mkd000 Wrote:I got this question correct but reviewing the question, I'm wondering how (A) can be considered a reversal. The first sentence of the stimulus says "most readers" so is it actually possible to call this sentence a conditional statement?

Please clarify! Thank you.


The first sentence of the stimulus says, "A novel cannot be of the highest quality unless most readers become emotionally engaged with the imaginary world it describes." You can diagram this as a conditional statement:
Novel is of the highest quality --> most readers become emotionally engaged
the contrapositive is:
~ most readers emotionally engaged --> ~ novel is of the highest quality.

Now, when you look at (A) the last part of this sentence says, "...only if the novel is of the highest quality". "Only if" indicates that the phrase after it is the necessary condition. So even though the first half of the sentence doesn't have a sufficient condition indicator, this latter half of the sentence shows that there is a necessary condition so the first half, "most readers become emotionally engaged" must be the sufficient condition.
Hence, you get the diagram:
most readers emotionally engaged --> novel is of the highest quality
As you can see from above, this is a reversal of the sentence in the stimulus.
 
daijob
Thanks Received: 0
Elle Woods
Elle Woods
 
Posts: 74
Joined: June 02nd, 2015
 
 
 

Re: Q12 - Critic: A novel cannot be

by daijob Sat Jul 25, 2015 9:42 am

Is the first sentence conclusion or the second one starting from "Thus...work" the conclusion?
If the first sentence is premise, is C connects two premises?

Thanks,
User avatar
 
ohthatpatrick
Thanks Received: 3808
Atticus Finch
Atticus Finch
 
Posts: 4661
Joined: April 01st, 2011
 
 
 

Re: Q12 - Critic: A novel cannot be

by ohthatpatrick Mon Jul 27, 2015 4:58 pm

Thus, therefore, hence, so, as a result, consequently = CONCLUSION

Because, since, after all, for = PREMISE

So you are correct, everything from "Thus .... work" is the conclusion. The idea following "since" is a premise, as is the first sentence.

The conclusion says that "SHIFTS detract from the merit of the work".

The 1st sentence speaks to "the merit of the work".

The 3rd idea (after "since") speaks to SHIFTS.

Since both terms in the conclusion are already mapped to ideas in the Evidence, the missing link is between the two parts of the evidence.

The missing link is between "whatever is connected to MERIT OF THE WORK in the 1st sentence" and "whatever is connected to SHIFTS" in the 3rd idea.

Another example of the same setup:
Playing golf doesn't involve any running.
If an activity doesn't raise your resting heart rate, it is not a real sport.
Thus, playing golf is not a real sport.

Missing link?



Activities that don't involve running don't raise your resting heart rate.
or
Anything that raises your resting heart rate involves running.
 
JuluK218
Thanks Received: 0
Vinny Gambini
Vinny Gambini
 
Posts: 1
Joined: September 19th, 2017
 
 
 

Re: Q12 - Critic: A novel cannot be

by JuluK218 Sun Oct 22, 2017 2:27 am

ohthatpatrick Wrote:Question Type:
Necessary Assumption

(B) Reversal. This is the same basic problem as A. "highest quality -> engaged imagination"


Why is answer choice B a mistaken reversal of the stimulus? Isn't it just a premise booster? It seems to be repeating the first sentence of the stimulus. Thanks!
User avatar
 
ohthatpatrick
Thanks Received: 3808
Atticus Finch
Atticus Finch
 
Posts: 4661
Joined: April 01st, 2011
 
 
 

Re: Q12 - Critic: A novel cannot be

by ohthatpatrick Wed Oct 25, 2017 2:44 pm

Thanks, that was a mistake. It is not a reversal. I'll go back and fix that.

(A) Emotionally engaged ---> highest quality
(B) High quality --> engages imagination

The first sentence was
Highest quality--> engages emotions

So it's not quite re-stating the first sentence
 
DevorahK415
Thanks Received: 0
Vinny Gambini
Vinny Gambini
 
Posts: 10
Joined: December 01st, 2019
 
 
 

Re: Q12 - Critic: A novel cannot be

by DevorahK415 Thu Jun 04, 2020 7:54 pm

Question about NA and SA: "assumption necessary" sounds like an NA --> negate to find the answer once down to 2 options.
But "properly drawn" sounds like an SA --> which would take us to contrapositive territory.

This is the second stem that uses both pieces of language tonight. Can I assume that NA language trumps SA language as it seems to be doing in all the answers on the forum? Thanks.
User avatar
 
smiller
Thanks Received: 73
Atticus Finch
Atticus Finch
 
Posts: 205
Joined: February 01st, 2013
 
 
 

Re: Q12 - Critic: A novel cannot be

by smiller Fri Jun 05, 2020 2:04 pm

Yep. It's good to be clear about this. When an LR question asks for an assumption that is necessary, it is definitely a Necessary assumption question. This applies even if there is other language in the question stem, like "properly drawn," that also appears in a Sufficient Assumption question stem. Necessary means necessary.

Watch for other words and phrases that mean the same as "necessary," such as "an assumption that is required" or "depends on assuming." These also indicate a Necessary Assumption question.

The weird exception to remember is a question that asks, "which of the following is assumed?" or "the argument assumes which of the following?" These question stems don't contain the word "necessary" or an equivalent word or phrase, but they are asking for a necessary assumption.

Sufficient Assumption questions stems follow this basic pattern: "if the following is assumed, then the conclusion follows logically." They will phrase this in different ways, and often use "properly drawn" or "properly inferred" rather than "follows logically," but the basic format and meaning of the question stem will match this pattern.
 
AnnaT620
Thanks Received: 0
Elle Woods
Elle Woods
 
Posts: 51
Joined: May 25th, 2020
 
 
 

Re: Q12 - Critic: A novel cannot be

by AnnaT620 Thu Jul 23, 2020 3:35 pm

Hi MP Team

I picked B here, after I tried to diagram this question:

High Quality --> Reader involved in Imaginary World
Shift in Narrative --> ~ Reader involved in Imaginary World

I eliminated C because of the "most", and tended to B because it was a conditional statement. When I diagrammed B I got High Quality --> Reader involved in Imaginary World. Negating that would then destroy the argument?

Where did I start to go wrong here? Have I diagrammed this incorrectly? Would you generally recommend diagramming this?

Thank you so much!
 
BarryM800
Thanks Received: 0
Elle Woods
Elle Woods
 
Posts: 64
Joined: March 08th, 2018
 
 
 

Re: Q12 - Critic: A novel cannot be

by BarryM800 Sun Dec 27, 2020 10:43 am

I'm confused by the explanations of this question. Pat says that the conclusion doesn't have any new guys, but "detract from the merit of the work" appears only in the conclusion, but not in the premises. He also states "the first sentence speaks to the merit of the work," but the first sentence only establishes "NOT most readers engaged → NOT highest quality." Apparently, the author also assumes "NOT highest quality → detract from the merit of work," or at least we have to assume this for the argument to work.

With the assumption stated in (C), we get the following logic chain: shifts -[m]→ most readers focus on the author → NOT most readers engaged → NOT highest quality → detract from the merit of the work.

The first link is established by "since such shifts tend to make most readers focus on the author," but this is a "most" statement. How can the argument draw an absolute conclusion (conditional: "shifts ... detract from the merit of the work"), based on a premise that has weaker logical force? Thanks!
 
Misti Duvall
Thanks Received: 13
Atticus Finch
Atticus Finch
 
Posts: 191
Joined: June 23rd, 2016
 
 
 

Re: Q12 - Critic: A novel cannot be

by Misti Duvall Sun Jan 10, 2021 7:41 pm

BarryM800 Wrote:I'm confused by the explanations of this question. Pat says that the conclusion doesn't have any new guys, but "detract from the merit of the work" appears only in the conclusion, but not in the premises. He also states "the first sentence speaks to the merit of the work," but the first sentence only establishes "NOT most readers engaged → NOT highest quality." Apparently, the author also assumes "NOT highest quality → detract from the merit of work," or at least we have to assume this for the argument to work.

With the assumption stated in (C), we get the following logic chain: shifts -[m]→ most readers focus on the author → NOT most readers engaged → NOT highest quality → detract from the merit of the work.

The first link is established by "since such shifts tend to make most readers focus on the author," but this is a "most" statement. How can the argument draw an absolute conclusion (conditional: "shifts ... detract from the merit of the work"), based on a premise that has weaker logical force? Thanks!



No worries! I think the key here is noting that this is a necessary assumption question, not a sufficient assumption question (see smiller's post above). Since it's necessary, the answer doesn't need to guarantee the conclusion; it just needs to be something the argument needs in order to work (think of a necessary assumption like a support beam). It's also ok to not fill every gap in the argument for a necessary assumption question, so even if you think there's a shift between detracts from merit and not highest quality (which I think is a fair argument), it's still ok.
LSAT Instructor | Manhattan Prep